Jamel is painting his room he determins that 1/2 gallons container of pain will cover 1/6 of a wall how many gallons of paint are needed for an entire wall (assuming there no doors or windows) the answer is not 120 gallons.



A pair of shoes originally cost $35 but they are on sale for 15% off what is the sale price of the shoes?

The answer is not 5.25



A community center is offspring a discount on swimming passes the regular cost for a swimming pass is 6:00 Jake, Lisa, and Manuel each buy a swimming pass at the community center after the discount the total cost for the 3 passes is $14.40 what is the discount the community center is offspring

A. 20%
B. 42%
C. 72%
D. 80%

D. Is not the answer!

Answers

Answer 1

Answer:

Step-by-step explanation:

(for the first question)

1/2 gallons cover 1/6th of the wall then

1-gallon covers 1/3rd of the wall

so 3 gallons cover one wall

(second question)

you have to calculate 85% of $35 because it is 15% percent off.

35*0.85=29.75

The sale price is $29.75.

(third question)

6*3=18

14.4*100/18

1440/18

80

100-80 = 20

The answer is A (20%)

Answer 2

Answer:

3 gallons of paint

$29.75

20%

Step-by-step explanation:

1. Let's break this down:

1/2 gallon of paint covers 1/6 of his wall.

This means that we have to multiply 1/2 by 6, as there would be 6 1/2 gallon sections of his wall.

1/2·6=3

So, Jamel needs 3 gallons of paint.

2. If a pair of shoes has an original price of $35 but it on sale for 15%, we have to first find how much it's now on sale for:

15/100·35

=0.15·35

=5.25

This is the how much it's off, so subtract 5.25 from 35

35-5.25=$29.75

The shoes have a sale price of $29.75

Even though you said this was wrong, you may have to put a dollar sign in front of it.

3. This is worded a little, but assuming:

1 swimming pass is $6, 3 people buy the swimming pass, and the total cost for the 3 passes in total is $14.40, we have to find out the discount.

So, originally, before the discount, the total amount for the 3 swimming passes would've been $18, but there's been a discount and now they only had to pay $14.40.

To solve, we do the following:

subtract the original price by the sale price

18-14.40=3.6

divide by the original price

3.6/18=0.2

multiply by 100 to get into percent

0.2x100=20%

This means that A is the correct choice.

Hope this helps! :)


Related Questions

Can someone help me ASAP please? It’s due tomorrow. Show work please!! I will give brainliest if it’s correct and has work.

Answers

The difference in the number of outcomes depending on the coins being replaced is B. 10 outcomes.

How to find the outcomes ?

For the first coin, there are 10 possible outcomes (any one of the 10 coins in the jar). For the second coin, there are again 10 possible outcomes, since the first coin is replaced and all 10 coins remain in the jar. Therefore, the total number of outcomes when two coins are selected with replacement is 10 x 10 = 100.

The number of outcomes when two coins are selected without replacement can be calculated as follows:

For the first coin, there are 10 possible outcomes (any one of the 10 coins in the jar). For the second coin, there are only 9 possible outcomes, since one coin has already been removed from the jar. Therefore, the total number of outcomes when two coins are selected without replacement is 10 x 9 = 90.

Difference is:

= 100 - 90

= 10 outcomes

Find out more on outcomes at https://brainly.com/question/1262944

#SPJ1

The legs of a right triangle measure 11.4 meters and 15.1 meters. To the nearest tenth, what is the measure of the smallest angle

Answers

The measure of the smallest angle is 37.1 degrees

Calculating the measure of the smallest angle

From the question, we have the following parameters that can be used in our computation:

The legs of a right triangle measure 11.4 meters and 15.1 meters

So, the measure of one of the acute angles is

tan(x) = 11.4/15.1

Evaluate

tan(x) = 0.7550

Take the arc tan of both sides

So, we have

x = 37.1

This means that the measure of the smallest angle is 37.1 degrees

Read more about right triangles at

https://brainly.com/question/2437195

#SPJ1

Select all of the statements that are true

The [9.7] = -9.7 because the distance from -9.7 to 0 on the number line is 9.7 units.

Numbers with the same absolute value are opposites because they are the same distance from each other.

The [7.1] = 7.1 because the distance from 7.1 to 0 on the number line is 7.1 units.

The [-8.4] = 8.4 because the distance from -8.4 to 8.4 on the number line is 0 units.

Numbers with the same absolute value are opposites because they are the same distance from 0 on the number line.

The [-12.5] = 12.5 because the distance from 12.5 to 0 on the number line is -12.5 units.

Answers

The true statements are Numbers with same absolute value are opposites because they are same distance from each other and from 0 on the number line. The |7.1| = 7.1. So, correct options are B, C and E.

b) Numbers with the same absolute value are opposites because they are the same distance from each other. This is true because absolute value is the distance from a number to zero on the number line, and if two numbers have the same distance from zero, then they must be equidistant from zero and therefore, they are opposite in sign.

c) The |7.1| = 7.1 because the distance from 7.1 to 0 on the number line is 7.1 units. This is true because the absolute value of a number is always positive, and it represents the distance of that number from zero on the number line.

d) The |-8.4| = 8.4 because the distance from -8.4 to 8.4 on the number line is 0 units. This is false, as the distance between -8.4 and 8.4 on the number line is 16.8 units. The correct value of the absolute value of -8.4 is 8.4.

e) Numbers with the same absolute value are opposites because they are the same distance from 0 on the number line. This is true because 0 is the midpoint of the number line, and if two numbers have the same distance from 0, then they must be equidistant from zero and therefore, they are opposite in sign.

Therefore, the correct statements are b, c, and e.

To learn more about distance/absolute value click on,

https://brainly.com/question/16901376

#SPJ1

what is the radius of a basketball if the volume is 11488.2 cm? round your answer the the nearest whole number. use 3.14 as π .

Answers

Answer:

The radius of the basketball is 20 cm.

Step-by-step explanation:

The formula for the volume of a sphere is V = (4/3)πr^3, where V is the volume and r is the radius.

We are given that the volume of the basketball is 11488.2 cm, so we can set up the equation:

11488.2 = (4/3)πr^3

Simplifying, we get:

(4/3)πr^3 = 11488.2

Dividing both sides by (4/3)π, we get:

r^3 = 11488.2 / (4/3)πr^3 = 7239.79

Taking the cube root of both sides, we get:

r ≈ 20

Rounding to the nearest whole number, the radius of the basketball is 20 cm.

Here is some information about 26 houses. A,b and c are all different numbers. Number of bedrooms:1,2,3,4,5. Number of houses:7,a,b,c,8. The median number of bedrooms is 3. 5 Work out a possible set of values for a,b and c

Answers

The possible set of values for a, b, and c could be: a=2, b=4, c=5.

Here is a possible set of values for a, b, and c,

- a = 2 (since there are 7 houses with 1-2 bedrooms and 8 houses in total, we know that there must be at least 1 more house with 1-2 bedrooms, which could be house a)
- b = 4 (since the median number of bedrooms is 3 and there are 7+1+1=9 houses total with either 1, 2, or 3 bedrooms, we know that the median house must have either 3 or 4 bedrooms. Since b must be different from a and c, we can assign it to 4)
- c = 5 (since there are only 3 houses left and we need to assign one to each remaining number of bedrooms, we can assign c to 5)

Therefore, a possible set of values for a, b, and c could be: a=2, b=4, c=5.

Learn more about "value":

https://brainly.com/question/843074

#SPJ11

-9 -7 -5 sequence name pls ​

Answers

The sequence would be -2.

This is the Arithmetic sequence.

A population has a proportion of 0. 62 and a standard deviation of sample proportions of 0. 8. A sample of size 40 was taken from this population. Determine the following probabilities. Illustrate each on the normal curve shown below each part.


a. ) The probability the sample has a proportion between 0. 5 and 0. 7



b. ) The probability the sample has a proportion within 5% of the population proportion



c. ) The probability that the sample has a proportion less than 0. 50



d. ) The probability that the sample has a proportion greater than 0. 80

Answers

The probability that a) the sample has a proportion between 0.5 and 0.7 is 0.780. b) The probability that the sample has a proportion within 5% is 0.819. c) The probability that the sample has a proportion less than 0.50 is 0.001. d) The probability that the sample has a proportion greater than 0.80 is 0.000.

a) To calculate this probability, we first need to standardize the interval (0.5, 0.7) using the formula: z = (p - P) / (σ / √(n))

where p is the sample b, P is the population proportion, σ is the standard deviation of sample proportions, and n is the sample size. Substituting the values, we get:

z1 = (0.5 - 0.62) / (0.8 / √(40)) = -2.24

z2 = (0.7 - 0.62) / (0.8 / √(40)) = 1.12

Using the standard normal table or calculator, the area between -2.24 and 1.12 is 0.780. Therefore, the probability that the sample has a proportion between 0.5 and 0.7 is 0.780.

b) The probability that the sample has a proportion within 5% of the population proportion is 0.819. We can find the range of sample proportions within 5% of the population proportion by adding and subtracting 5% of the population proportion from it, which gives: P ± 0.05P = 0.62 ± 0.031

The interval (0.589, 0.651) represents the range of sample proportions within 5% of the population proportion. To calculate the probability that the sample proportion falls within this interval, we standardize it using the formula above and find the area under the standard normal curve between -1.55 and 1.55, which is 0.819.

c) The probability that the sample has a proportion less than 0.50 is 0.001. To calculate this probability, we standardize the value of 0.50 using the formula above and find the area to the left of the resulting z-score, which is: z = (0.50 - 0.62) / (0.8 / √(40)) = -4.46

Using the standard normal table or calculator, the area to the left of -4.46 is 0.001. Therefore, the probability that the sample has a proportion less than 0.50 is 0.001.

d) The probability that the sample has a proportion greater than 0.80 is 0.000. To calculate this probability, we standardize the value of 0.80 using the formula above and find the area to the right of the resulting z-score, which is: z = (0.80 - 0.62) / (0.8 / √(40)) = 5.60

Using the standard normal table or calculator, the area to the right of 5.60 is very close to 0.000. Therefore, the probability that the sample has a proportion greater than 0.80 is 0.000.

To know more about probability, refer here:

https://brainly.com/question/16988487#

#SPJ11

If a 35 N block is resting on a steel table with a coefficient of




static friction Hs = 0,40, then what minimum force is required to




move the block.

Answers

The minimum force required to move a block of 35 N resting on a steel table with a coefficient of static friction of 0.40 is 14 N.

Friction refers to the force that resists the motion and thus the force acts in the opposite direction of the force applied.

There are the following types of friction:

1. Static Friction

2. Limiting Friction

3. Kinetic Friction

F = μN

where μ is the coefficient of friction

N is the Normal Force

When the object is resting on a table, Normal force is the weight.

N = 35 N

μ = 0.40

F = 0.4 * 35

= 14 N

Learn more about Friction:

https://brainly.com/question/24338873

#SPJ4

how do i find the inverse

Answers

Step-by-step explanation:

To solve for inverse, utilize the following steps.

Step 1: let f(x)=y so we get

[tex]y = \sqrt{x - 6} + 5[/tex]

Step 2: Swap y and x

[tex]x = \sqrt{y - 6} + 5[/tex]

Solve for y.

[tex]x - 5 = \sqrt{y - 6} [/tex]

[tex](x - 5) { }^{2} + 6 = y[/tex]

Step 4: Let y =f^-1(x)

[tex](x - 5) {}^{2} + 6 = f {}^{ - 1} (x)[/tex]

Answer: [tex]f^{-1}(x) =[/tex] x²-10x+19

Step-by-step explanation:

Let's replace f(x) for y for now.

[tex]y=\sqrt{x-6}+5[/tex]

To find inverse.  make your y into x, and your x into y

[tex]x=\sqrt{y-6}+5[/tex]        >Now you solve for y.  subtract 5 from both sides

[tex]x-5=\sqrt{y-6}[/tex]        >Square both sides to get rid of root

[tex](x-5)^{2} =(\sqrt{y-6})^{2}[/tex]     >drop root and square (x-5)

(x-5)(x-5) = y-6              >FOIL

x²-5x-5x+25 = y-6        > combine like terms

x²-10x+25 = y-6            >add 6 to both sides

x²-10x+19=y            > this is your inverse now put the y into inverse form

[tex]f^{-1}(x) =[/tex] x²-10x+19

The mass of the Rock of Gibraltar is 1. 78 ⋅ 1012 kilograms. The mass of the Antarctic iceberg is 4. 55 ⋅ 1013 kilograms. Approximately how many more kilograms is the mass of the Antarctic iceberg than the mass of the Rock of Gibraltar? Show your work and write your answer in scientific notation

Answers

The mass of the Antarctic iceberg is approximately 2.56 × 10¹more kilograms than the mass of the Rock of Gibraltar.

To find out, we can subtract the mass of the Rock of Gibraltar from the mass of the Antarctic iceberg:

4.55 × 10¹³ kg - 1.78 × 10¹² kg = 4.37 × 10¹³ kg

Therefore, the mass of the Antarctic iceberg is about 2.56 × 10¹ (or 25.6) times greater than the mass of the Rock of Gibraltar.

This is because the mass of the Antarctic iceberg is much larger than the mass of the Rock of Gibraltar, as it is a massive block of ice floating in the ocean while the Rock of Gibraltar is a solid rock formation on land.

To know more about Antarctic iceberg click on below link:

https://brainly.com/question/28864948#

#SPJ11

Taylor would like to have a karaoke deejay at her graduation party. her three sisters volunteered to split the cost of hiring the deejay. they need to rent a tent for $45 and a microphone system for $60 and then pay the deejay $30 an hour for four hours. how much do each of the sisters owe?

write out all the work used to determine the answer to the question.

Answers

Each of the three sisters owes $75 to cover the cost of hiring the karaoke deejay for Taylor's graduation party.

To determine how much each sister owes, we need to first calculate the total cost of the party and then divide that cost by three, since there are three sisters splitting the cost.

1. Tent rental: $45
2. Microphone system: $60
3. Deejay cost: $30/hour × 4 hours = $120

Now, we'll add these costs together to find the total cost:
Total cost = $45 (tent) + $60 (microphone) + $120 (deejay) = $225

Finally, we'll divide the total cost by the number of sisters (3) to find out how much each sister owes:
Amount owed per sister = $225 (total cost) ÷ 3 (sisters) = $75

So, each sister owes $75 for the karaoke deejay at Taylor's graduation party.

More on cost: https://brainly.com/question/13345756

#SPJ11

Here are the numbers of calls received at a customer support service during 8 randomly chosen, hour-long intervals.
9, 14, 23, 14, 19, 9,5,7
Send data to calculator
(a) What is the median of this data set? If your answer is not 0
an integer, round your answer to one decimal place.
(b) What is the mean of this data set? If your answer is not an
integer, round your answer to one decimal place.
(c) How many modes does the data set have, and what are
their values? Indicate the number of modes by clicking in the
appropriate circle, and then indicate the value(s) of the
mode(s), if applicable.
0
OO
zero modes
O one mode: 0
two modes:
and

Answers

a) The median of the dataset is: 11.5

b) The mean of the dataset is: 12.5

c) The mode of the dataset is: 9 and 14

How to find the mean, median or mode?

The term average mean is defined as the finding of the average of a sample data. Thus, the average is finding the central value in math, which tells us that mean is finding the central value in statistics.

The numbers arranged in ascending order is:

5, 7, 9, 9, 14, 14, 19, 23

a) The median is defined as the middle term of the distribution when arranged in ascending or descending order. Thus, the median here is:

(9 + 14)/2 = 11.5

b) The mean of the data is expressed as:

(5 + 7 + 9 + 9 + 14 + 14 + 19 + 23)/8

= 12.5

c) The mode is the most frequently occurring term in the data.

In this case, the mode is 9 and 14

Read more about mean, median or mode at: https://brainly.com/question/14532771

#SPJ1

Please solve, I do rate!
Given: (x is number of items) Demand function: d(x) = 588.7 – 0.4x2 Supply function: 8(x) = 0.322 2 Find the equilibrium quantity: Find the producers surplus at the equilibrium quantity:

Answers

The equilibrium quantity is approximately 34.47 items and the producer surplus at the equilibrium quantity is approximately 396.11.

How to find equilibrium quantity and producer surplus?

To find the equilibrium quantity, we need to find the quantity at which the demand and supply functions are equal:

Demand function: d(x) = 588.7 – 0.4x^2

Supply function: s(x) = 8(x) = 0.322

Setting these two functions equal to each other, we get:

588.7 – 0.4x^2 = 0.322x

Simplifying this equation, we get:

0.4x^2 + 0.322x - 588.7 = 0

Using the quadratic formula, we get:

x = (-0.322 ± √(0.322^2 + 40.4588.7)) / (2*0.4)

x ≈ 34.47 or x ≈ -43.67

Since we cannot have a negative quantity, the equilibrium quantity is approximately 34.47 items.

To find the producer surplus at the equilibrium quantity, we need to calculate the area between the supply curve and the equilibrium price, which is the price that corresponds to the equilibrium quantity. We can find the equilibrium price by plugging the equilibrium quantity into either the demand or supply function:

s(34.47) = 8(34.47) = 11.58

So the equilibrium price is approximately 11.58.

Now we can find the producer surplus by integrating the supply function from 0 to the equilibrium quantity, and subtracting the result from the area of a rectangle with height equal to the equilibrium price and width equal to the equilibrium quantity. The formula for producer surplus is:

Producer Surplus = (Equilibrium Price * Equilibrium Quantity) - ∫[0, Equilibrium Quantity] Supply Function dx

Plugging in the values we found, we get:

Producer Surplus = (11.58 * 34.47) - ∫[0, 34.47] 0.322 dx

Integrating the supply function, we get:

∫[0, 34.47] 0.322 dx = 0.322 * 34.47 ≈ 11.10

So the producer surplus is:

Producer Surplus ≈ (11.58 * 34.47) - 11.10 ≈ 396.11

Therefore, the equilibrium quantity is approximately 34.47 items, and the producer surplus at the equilibrium quantity is approximately 396.11.

Learn more about equilibrium quantity and producer surplus.

brainly.com/question/24096086

#SPJ11

Victoria will deposit $2000 in an account that earns 5% simple interest every year. Her friend Corbin will deposit $1800 in an account that earns 9% interest compounded annually. The deposits are made on the same day, and no additional money will be deposited or withdrawn from the accounts. Which statement about the balances of Victoria and Corbin's accounts at the end of 3 years is true?

Answers

Corbin's account will have a higher balance than Victoria's account at the end of 3 years" is true.

How to calculate account balance at the end of 3 years?

To calculate the balance at the end of 3 years, we can use the simple interest formula for Victoria's account and the compound interest formula for Corbin's account.

For Victoria's account:

Simple interest = P * r * t

= 2000 * 0.05 * 3

= $300

Balance after 3 years = P + Simple interest

= 2000 + 300

= $2300

For Corbin's account:

Balance after 3 years = [tex]P * (1 + r)^t[/tex]

= 1800 * (1 + 0.09)³

= $2401.40

Therefore, the statement "Corbin's account will have a higher balance than Victoria's account at the end of 3 years" is true.

Learn more about balance

brainly.com/question/22841923

#SPJ11

1) You want your savings account to have a total of $23,000 in it within 5 years. If you invest your money in an account that pays 6.8% interest compounded continuously, how much money must you have in your account now? 2) You buy a brand new Audi R8 for $148,700 before taxes. If the car depreciates at a rate of 8%, how much will it be worth in 5 years?

Answers

After 5 years with 8% depreciation, the Audi R8's value will be around $81,249.36.

To determine how much money you must have in your account now, you can use the formula A = Pe^(rt), where A is the final amount, P is the principal (the initial amount invested), e is the constant 2.71828, r is the annual interest rate expressed as a decimal, and t is the time in years. We will calculate using this formula.Plugging in the given values, we get:
A = $23,000
r = 0.068 (6.8% expressed as a decimal)
t = 5 years
So, $23,000 = P*e^(0.068*5)
Solving for P, we get:
P = $16,376.59
Therefore, you must have $16,376.59 in your account now to reach your goal of $23,000 in 5 years with 6.8% continuous compounding interest. To determine how much the Audi R8 will be worth in 5 years, you can use the formula A = P(1 - r)^t, where A is the final amount, P is the initial amount, r is the annual depreciation rate expressed as a decimal, and t is the time in years. Plugging in the given values, we get:
P = $148,700
r = 0.08 (8% expressed as a decimal)
t = 5 years
So, A = $148,700*(1 - 0.08)^5
Simplifying, we get:
A = $81,249.36
Therefore, the Audi R8 will be worth approximately $81,249.36 in 5 years with 8% depreciation.

Learn More About Depriciation: https://brainly.com/question/24279065

#SPJ11

A student wants to estimate the mean bowling score for all bowlers in a particular bowling league. fifty scores are randomly selected from the league with a
sample mean was 186 with a standard deviation of 22. assume normality.
5. construct a 95% confidence interval for the mean score for all bowlers in the league.
(179.75, 192.25
(177.66, 194.34)
(180.78, 191.22)
(163.83, 208.17)
(179.9, 192.1)

Answers

The 95% confidence interval for the mean score for all bowlers in the league is option (E) (179.9, 192.1).

To construct a 95% confidence interval for the mean score for all bowlers in the league, we can use the formula:

CI = X ± z* (σ/√n)

where X is the sample mean, σ is the population standard deviation (unknown), n is the sample size, and z* is the critical value for the desired confidence level (95% in this case).

Since the sample size is 50, we can assume that the population standard deviation is approximately equal to the sample standard deviation, which is 22. The critical value for a 95% confidence interval with a two-tailed test is 1.96.

Substituting the values, we get:

CI = 186 ± 1.96 (22/√50)

  = 186 ± 6.44

  = (179.56, 192.44)

Therefore, the answer is (B) (177.66, 194.34), which is the closest to the calculated confidence interval.

Learn more about "confidence interval":

https://brainly.com/question/15712887

#SPJ11

Unit 7: Right Triangles & Trigonometry Homework 4: Trigonometry Ratios & Finding Missing Sides #13

Answers

The value of sides are KL=5.34, JK=16.434, JL=17.29 and ML=22.25.

∵ ΔJLM is a right triangle, as ∠MJL=90°

tan(∠JML)= JL/JM            [∵ tan∅=perpendicular/hypotenuse]

⇒ tan(51°)=JL/14

⇒ JL=14×tan(51°)

       = 14×1.23

       = 17.29

JL=17.29

Again, ΔJKL is a right triangle, with ∠JKL=90°

cos(∠JLK)=KL/JL              [∵ cos∅=base/hypotenuse]

⇒cos(72°)= KL/17.29

⇒KL=17.29×cos(72°)

       = 17.29×0.309

        = 5.34

KL=5.34

Hence, the value of KL is 5.34.

Also, tan(∠JLK)=KJ/KL

⇒tan(72°)=JK/5.34

⇒JK=5.34×tan(72°)

       = 5.34×3.077

       = 16.434

JK=16.434

And, cos(∠JML)=JM/ML

⇒cos(51°)=14/ML

⇒ML=14/cos(51°)

        =14/.629

        =22.25

ML=22.25

Hence, the value of sides are KL=5.34, JK=16.434, JL=17.29 and ML=22.25.

For more problems on trigonometry,

https://brainly.com/question/13729598

https://brainly.com/question/25618616

Omar Cuts A Piece Of Wrapping Paper with the shape and dimensions as shown .Find the area of the wrapping paper. Round your answer to the nearest tenth if needed

Answers

The area of the wrapping paper would be = 72.5in².

How to calculate the area of the wrapping paper?

To calculate the area of the wrapping paper, the figure is first divided into two leading to the formation of a triangle and a rectangle.

For the triangle, the formula use to calculate it's area is given as follows;

Area = 1/2 base × height

base = 15-10 = 5 in

height = 9-4 = 5 in

area = 1/2×5 × 5

= 25/2 = 12.5 in²

Area of a rectangle = length× width

width = 4 in

length = 15 in

area = 4×15 = 60in²

Therefore the area of the wrapping paper = 12.5+60 = 72.5in²

Learn more about area here:

https://brainly.com/question/28470545

#SPJ1

A block of wood measures 6. 5 inches by 1. 5 inches by 8 inches. What is the volume of the block of wood?
Type your answer with cubic inches

Answers

The volume of the block of wood is 78 cubic inches.

What is cube?

A cube is a three-dimensional geometric shape that has six equal square faces, 12 equal edges, and eight vertices (corners). All the angles between the faces and edges of a cube are right angles (90 degrees), and all the edges are of equal length. A cube is a special type of rectangular prism where all the sides are equal in length, making it a regular polyhedron.

To find the volume of the block of wood, you need to multiply its length, width, and height together.

Volume = length x width x height

Volume = 6.5 inches x 1.5 inches x 8 inches

Volume = 78 cubic inches

Therefore, the volume of the block of wood is 78 cubic inches.

To learn more about cubes from the iven link:

https://brainly.com/question/28134860

#SPJ4

Instructors led an exercise class from a raised rectangular platform at the front of the room. The width of the platform is (x+4) meters long and the area of the rectangular platform is 3x^2+10x−8. Find the length of the platform

Answers

Length of the platform at the front of the room whose area is 3x² + 10x - 8 and width is (x+4) m is (3x - 2) m

Area of the rectangular platform = 3x² + 10x - 8

Width of the rectangular platform = x+4

Area = length × width

Length = area/width

Length = [tex]\frac{3x^{2} + 10x - 8}{x+4}[/tex]

By splitting the middle term we get

Length = [tex]\frac{3x^{2} + 12x -2x -8 }{x+4}[/tex]

By taking common we get

Length = [tex]\frac{3x(x+4) - 2(x+4)}{x+4}[/tex]

By taking x+4 common we get

Length = [tex]\frac{(3x-2)(x+4)}{x+4}[/tex]

Cutting the x+4 from denominator and numerator we get

Length = 3x-2

Length of the platform at the front of room is 3x-2

To know more about area click here :

https://brainly.com/question/27683633

#SPJ4

Select the statement that best describes the expression 4+3x
A. 4 plus 3plus x
B. The sum of 4 and 3
C. The product of 4 and 3x
D. 4 plus 3 times x

Answers

The correct option is D, the statement that best describes the expression 4+3x means "4 plus 3 times x".

An expression is a combination of numbers, symbols, and/or variables that represents a mathematical or logical statement. It can be as simple as a single number or letter, or as complex as a series of operations that involve multiple variables and functions. Expressions can be used to represent equations, inequalities, functions, and other mathematical concepts. They can be evaluated to produce a numerical value or a boolean value (true or false) depending on the values of the variables involved.

Expressions are used to represent calculations or logical conditions. They can be used to assign values to variables, manipulate data, and control the flow of a program. expressions are a fundamental concept in both mathematics and computer science, and play a critical role in solving problems and building complex systems.

To learn more about Expression visit here:

brainly.com/question/29404073

#SPJ4

Let F(X) = - 8 - x^2, find the following:
(f(7) - f(3))/ 7 -3

Answers

A relation is a set of ordered pairs that define the relationship between two sets. And, a function is a relation in which each element of the domain is connected to a single element of the codomain. The evaluated function is -10.

To find the expression (f(7) - f(3))/ 7 -3, we need to first find f(7) and f(3).

Using the given function F(X) = - 8 - x^2, we can find:

f(7) = -8 - 7^2 = -57

f(3) = -8 - 3^2 = -17

Now, we can substitute these values into the expression:

(f(7) - f(3))/ 7 -3 = (-57 - (-17))/ (7-3) = -40/4 = -10

Therefore, the answer is -10.
Visit here to learn more about point function:

brainly.com/question/1413672

#SPJ11

If x = yand y = z, which statement must be true?

O A. -x=-z

O B. z=x

O c. x=z

O D. -x=z

Answers

Answer:

The answer is C. x=z

Step-by-step explanation:

The correct answer is C. x=z.

Since x = y and y = z, then x = z. This is the transitive property of equality.

Here is a more detailed explanation:

The transitive property of equality states that if a = b and b = c, then a = c.

In this case, x = y and y = z. Therefore, x = z.

WILL MARK BRAINLIEST!

Answers

a. The property damage insurance covers the damage to the fence.

How to calculate the insurance

b. The insurance company will pay $7,000 - $1,000 = $6,000 for the fence damage.

c. The insurance company will pay $24,000 for the bus damage and $2,100 - $1,000 = $1,100 for the car damage.

d. The collision insurance policy covers the damage to Stewart's car.

e. The insurance company will pay $3,600 - $1,000 + $2,100 - $1,000 = $3,700 for the damage to the car.

Learn more about insurance on

https://brainly.com/question/25855858

#SPJ1

Two sisters, working together can clean the house in 3 hours. The older sister works 3 times faster than the younger sister when cleaning the house. How long will it take the younger sister to finish the same job by herself? Type just the number don't include words

Answers

The time taken by the younger sister to finish the same work by herself is 12 hours.

To solve the problem of how long it will take the younger sister to finish the job by herself, let's use the following terms:

1. Older sister's work rate = O
2. Younger sister's work rate = Y
3. Time taken by the younger sister alone = T

Given that the older sister works 3 times faster than the younger sister, we have:  O = 3Y.

Also, the sisters together can finish the job in 3 hours. Therefore, their combined work rate is equal to completing 1/3 of the job per hour. So,

O+Y=1/3.

Now, we can substitute O with 3Y:  3Y+Y=1/3. Combine the terms and simplify:

4Y=1/3

Now, solve for Y:

Y=1/12

Since Y is the work rate of the younger sister, to find the time it takes for her to complete the job alone (T), we can use the following formula:

Work rate × Time = 1 job.

So, Y × T = 1.

Substitute Y with  1/12:

[tex]\frac{1}{12} \times T=1[/tex]

Now, solve for T:

T = 12.

Therefore, it will take the younger sister 12 hours to finish the job by herself.

To know more about work refer here:

https://brainly.com/question/8632803#

#SPJ11

Which expression had a value less than 1

Answers

Step-by-step explanation:

[tex] - \infty \: and \: 0[/tex]

or

[tex]x \leqslant 1[/tex]

Bet you can’t solve this

Answers

Answer: The answer is (A

Step-by-step explanation:

The answer isB because A is constant, C is irrelevant, and D is dependent.

The chamber of commerce for a beach town asked a random sample of city dwellers, "Would you like to live at the beach?" Based on this survey, the 95% confidence interval for the population proportion of city dwellers who would like to live at the beach is (0. 56, 0. 62)

Answers

The 95% confidence interval for the population proportion of city dwellers who would like to live at the beach is estimated to be between 0.56 and 0.62.

How to find the sample size of the random survey?

A statistical inference  is a range of values within which the true value of a population parameter, such as the proportion of city dwellers who would like to live at the beach, is likely to fall with a certain level of confidence. In this case, the chamber of commerce for a beach town asked a random sample of city dwellers whether they would like to live at the beach, and based on the survey results, they constructed a 95% confidence interval for the population proportion.

The 95% confidence interval they obtained was (0.56, 0.62). This means that if they were to repeat their survey many times and construct a confidence interval each time, approximately 95% of those intervals would contain the true value of the population proportion.

In practical terms, this means that the chamber of commerce can be reasonably confident that the true proportion of city dwellers who would like to live at the beach falls somewhere between 0.56 and 0.62. It also suggests that the proportion of city dwellers who would like to live at the beach is relatively high, with more than half of the sample expressing a desire to do so. However, it is important to keep in mind that this confidence interval is based on a sample of city dwellers, and the true population proportion could differ from this estimate.

Learn more about statistical inference

brainly.com/question/3640262

#SPJ11

For questions 1,2, and 3 find intervals of positive and negative r values. 1. r= 1 - 2 cos θ 2. r= 5 sin (3θ) 3. r= 1 - 5 sin θ

Answers

r has negative values when 2 cos θ > 1, and positive values otherwise.

r has negative values when 3θ is in the second or third quadrant, and positive values otherwise.

r has negative values when sin θ > 1/5, and positive values otherwise.

To find the intervals of positive and negative r values, we need to look at the cosine function. Since the cosine function has a maximum value of 1, we have r = 1 - 2 cos θ ≥ -1. Solving for cos θ, we get 2 cos θ ≤ 2, which means that r is negative when 2 cos θ > 1 and positive otherwise.

We can rewrite the polar equation r = 5 sin (3θ) as r = 5(sin θ)(cos^2 θ)(3)^(1/2). This equation is negative when sin θ is negative, which happens in the second and third quadrants. Therefore, r is negative when 3θ is in the second or third quadrant and positive otherwise.

Similarly, we can rewrite the polar equation r = 1 - 5 sin θ as r = 5(cos θ)(sin(π/2 - θ)). This equation is negative when sin(π/2 - θ) is negative, which happens when θ is in the second and third quadrants. Therefore, r is negative when sin θ > 1/5, and positive otherwise.

For more questions like Function click the link below:

https://brainly.com/question/16008229

#SPJ11

Briefly discuss the difference between indefinite integral and definite integral. Give an example to provide emphasis. *​

Answers

A definite integral is defined as the signed area under a function between certain limits (bounds) of integration.

An indefinite integral represents the family of antiderivatives of a function and is also known as its general integral or antiderivative.

The difference between the integrals

An indefinite integral represents the family of antiderivatives of a function and is also known as its general integral or antiderivative. An indefinite integral does not have specific limits of integration; its result includes a constant of integration (usually denoted +C), which accounts for all possible constant shifts within its antiderivative.

A definite integral is defined as the signed area under a function between certain limits (bounds) of integration. The real number that represents its net area between it and x-axis during an interval.

Read more on indefinite integral and definite integral here:https://brainly.com/question/27419605

#SPJ4

Other Questions
Suggest at least two ways that such a person might use a handheld computer to work more efficiently as a manager in a supermarket professionals. in the figure shown, the darker square is removed. Divide the remaining figure into two rectangles Research topic internal resistance to apartheid in the 1980s A quadratic expression has x + 4 and 4x + 9 as its linear factors. Between which values ofx can a zero of the associated quadratic function be found? Who are the opposing forces in the conflict in this passage? G What two carbonyl compounds are needed to synthesize each of the following compounds, using a Robinson annulation? Part A A line-angle formula shows a ring with six vertices and a double bond between the second (clockwise) and the third vertices. An oxygen atom is double-bonded to the first vertex. A COCH3 group, whose first (from left to right) carbon is double-bonded to an oxygen atom, is attached to the fourth vertex Frank wants to find the area enclosed by the figure at the right the figure on each side has semicircles of a 40 meter by 40 meter square. Find the area by the figure use 3. 14 for pi Suppose that you are gambling at a casino. Every day you play at a slot machine, and your goal is to minimize your losses. We model this as the experts problem. Every day you must take the advice of one of n experts (i. E. A slot machine). At the end of each day t, if you take advice from expert i, the advice costs you some c t i in [0, 1]. You want to minimize the regret R, defined as: Toad and Toadette just had their first little toadstool! Toad's family is known to be purebred dominant for red spots on their white cap. Everyone was shocked when Little Toad was born with a white cap with white spots instead of red. Toadette is very upset as she thinks the Mushroom Kingdom Hospital accidentally switched babies. Is this true? Did the hospital really switch babies? Choose either "yes" or "no" and defend your answer. The difference of two rational numbers is 17/28,if the small rational number is -9/14 find the other Marcus is taking part in a charity run. He has received $250 in fixed pledges, and he will receive $25 more in pledges for each mile he runs. Write an equation for the amount of money P Marcus will earn in terms of the distance d he runs, measured in miles Need help with number 1!! Pleaseee Fred Rodriguez and Alma Morton are running for an open U.S. Senate seat. During a debate, they had this exchange:"I believe that people should take responsibility for their actions," Fred said. "If they are going to hurt others then they should pay for their actions. Unsupervised children are to blame for many crimes. Their parents should be held accountable.""Many parents in our community work full time," Alma replied. "They don't have enough income to pay a child-care provider. And now you're suggesting that they should be arrested for crimes their children commit? How are they supposed to feed their families if they are in jail? Shame on you for suggesting such a thing! I support the Families Now tax that provides funds for child services.""The last thing we need is another tax," Fred said. "Local nonprofit agencies should help children instead."Which candidates statement implies he or she has sympathy for the problems of working families and would tend to support their platform? Other than the usa, what country produces the most energy via nuclear power?. which two accounts have the most liquidity?a. under the mattress and savings account b. under the mattress and CD (certification of deposit)c. under the mattress and savings bond.d. money market and savings bonds. How did the use of machine guns, airplanes, submarines, poison gas, tanks, and other technologies change the way wars have been fought? Write 1 paragraph containing at least 3 details/pieces of evidence to support your answer Which tissue type forms binding,supporting, and transporting structuresand usually contains collagen forstrength?A. Epithelial tissueB. Muscular tissueC. Connective tissueD. Nervous tissue Which of the statements most likely describes an example of chemical weathering. A. the widening of cracks in rock by tree roots B. the rusting of iron rich rocks C. the expansion and contraction of rocks as tempertures change D. the scouring of rocks by windblown sand Stimulation of the reticular formation will cause a:. give two examples of services that are provided by municipalities which are meant to ensure safe and healthy living environments to communities and state one reason why most South Africans still live under filthy condition